Soluções Física - Semana 156

Escrito por Akira Ito e Gabriel Hemétrio

Iniciante

Assunto abordado

Gravitação e Lei de Newton

[collapse]
Solução

Com as informações do enunciado, podemos montar a figura:

Logo, para que suas distâncias permaneçam inalteradas, as estrelas devem orbitar o CM, tal que:

F_{res} = 2F\cos 30^{\circ} = \dfrac{Mv^2}{\dfrac{d}{2\cos 30^{\circ} }}


\dfrac{2GM^2 \cos 30^{\circ}}{d^2} = \dfrac{2Mv^2 \cos 30^{\circ}}{d}


\dfrac{GM}{d} = v^2


\boxed{v = \sqrt{\dfrac{GM}{d}}}

[collapse]
Gabarito

\boxed{v = \sqrt{\dfrac{GM}{d}}}

[collapse]

Intermediário

Assunto abordado

Termodinâmica, Gravitação e Oscilações

[collapse]
Solução

a) No equilíbrio, a pressão interna deve equilibrar a força gravitacional. A força devido à pressão do gás é:

F=p_0A

Em que A=4\pi R_0^2, isto é, a área da esfera. A força gravitacional devido a um corpo esférico homogêneo é:

F_g=\dfrac{GMm}{R^2_0}

Logo, a condição de equilíbrio é:

\boxed{\dfrac{GMm}{R^2_0}=4\pi R_0^2 p_0}

b) Aplicando a segunda lei de Newton para a casca esférica:

4\pi R^2p-\dfrac{GMm}{R^2}=ma_R

\left(\dfrac{4\pi}{m} \right)pR^2- \dfrac{\left(GM\right)}{R^2} =a_R

Logo:

\boxed{A=4\pi/m} e \boxed{B=GM}

c) Para processos adiabáticos, vale a relação:

pV^\gamma=k

Em que p e V são a pressão e o volume do gás, respectivamente, e k é uma constante. Logo:

p_0V_0^\gamma=pV^\gamma

Mas o volume de uma esfera é V=4\pi R^2/3, assim:

\boxed{p_0R_0^{3\gamma}=pR^{3\gamma}}

d) Agora que temos uma relação entre a pressão e o raio, podemos usar o resultado do item c) no item b). Logo:

 4\pi R^2 p_0 \left( \dfrac{R_0}{R} \right)^{3\gamma}-\dfrac{GMm}{R^2}=ma_R

Vamos supor que, depois que o cometa passou, o raio aumentou em  \Delta R. Assim, aplicando a relação  R=R_0+\Delta R:

 4\pi R_0^{3\gamma} p_0 \left(R_0+\Delta R\right)^{2-3\gamma}-GMm(R_0+\Delta R)^{-2}=ma_R

 4\pi R_0^2 p_0 \left( 1+(2-3\gamma)\dfrac{\Delta R}{R_0} \right) -GMmR_0^{-2}\left( 1-2\dfrac{\Delta R}{R_0} \right)=ma_R

Note que, na segunda linha, usamos a aproximação de Taylor fornecida no começo do problema, já que a razão  \Delta R/R_0 é muito pequena. Além disso, é importante lembrar do resultado encontrado no item a), GMm/R^2_0=4\pi R_0^2 p_0 , para fazer algumas simplificações:

 -\dfrac{GMm}{R_0^3}(3\gamma-4)\Delta R =ma_R

Para que o movimento seja um MHS, a força resultante (lado esquerdo da equação) deve ser da forma  F=-k\Delta R, em que k é uma constante  positiva. Portanto:

\boxed{\gamma  data-recalc-dims= \dfrac{4}{3}}" />

e) Usando o valor de \gamma=5/3 na expressão do item d):

 -\dfrac{GMm}{R_0^3}\Delta R =ma_R

Logo, a frequência angular do movimento é:

 \omega^2=\dfrac{GM}{R_0^3}

Então o período vale:

 \boxed{T=2\pi \sqrt{\dfrac{R_0^3}{GM}}}

[collapse]
Gabarito

a)

\boxed{\dfrac{GMm}{R^2_0}=4\pi R_0^2 p_0}

b)

\boxed{A=4\pi/m} e \boxed{B=GM}

c)

\boxed{p_0R_0^{3\gamma}=pR^{3\gamma}}

d)

\boxed{\gamma  data-recalc-dims= \dfrac{4}{3}}" />

e)

 \boxed{T=2\pi \sqrt{\dfrac{R_0^3}{GM}}}

[collapse]

Avançado

Assunto abordado

Termodinâmica e Gravitação

[collapse]
Solução

a) Primeiramente, podemos encontrar a pressão exercida por um sistema na parede de uma caixa:

 P = \frac{\Delta p}{A \Delta t}

mas:

dp = 2mv\cos \theta dN


dp = (2mv\cos \theta)(A v\cos \theta nf(v)dv \frac{1}{2} \sin \theta d\theta dt)


\Delta p = \int_{0}^{\infty} \int_{0}^{\frac{\pi}{2}} \int_{0}^{\Delta t} (2mv\cos \theta)(A v\cos \theta nf(v)dv \frac{1}{2} \sin \theta d\theta dt)


\Delta p = \frac{nmA \langle v^2 \rangle \Delta t}{3}

Finalmente, utilizando a relação para pressão:

P = \frac{nm \langle v^2 \rangle }{3}

Para um gás de fótons  \langle v^2 \rangle = c^2 , tal que obtemos a equação de estado do tipoP = \dfrac{\rho c^2}{3}, ou seja:

\boxed{P = \dfrac{u}{3}}

b)  Escrevendo a Primeira Lei da Termodinâmica para o sistema:

TdS = dU + pdV


diferenciando:

T \left( \dfrac{\partial S}{\partial V} \right)_T = \left( \dfrac{\partial U}{\partial V} \right)_T + p


utilizando uma das relações de Maxwell:

T \left( \dfrac{\partial p}{\partial T} \right)_V = \left( \dfrac{\partial U}{\partial V} \right)_T + p


Logo;

 T \left( \dfrac{\partial p}{\partial T} \right)_V = u + p


Agora, utilizando a equação de estados para um fóton P = \dfrac{u}{3}, encontramos:

 T \left( \dfrac{\partial u}{3\partial T} \right)_V = \dfrac{4u}{3}


 \boxed{ \left( \dfrac{\partial u}{\partial T} \right)_V = \dfrac{4u}{T}}

c)  Integrando a equação anterior:

 \int \dfrac{du}{u} = 4 \int \dfrac{dT}{T}


 \ln u = 4 \ln T + C


 u = aT^4


Para normalizar a função, note que a intensidade é dada por:

I = \Phi U


I = \dfrac{\eta c U}{4} = \dfrac{uc}{4}


Mas, sabemos que:

I = \sigma T^4


Logo:

 \dfrac{ac}{4} = \sigma


 a = \dfrac{4 \sigma}{c}


Então:

u = \dfrac{4 \sigma T^4}{c}


E, por fim:

\boxed{P = \dfrac{4 \sigma T^4}{3c}}

d)  Nesse caso, haverá, na superfície da estrelas, pressões devido à radiação e ao gás ideal e essas serão responsáveis pelo equilíbrio hidrostático da parcela adicional de poeira. Sabemos, pela equação de equilíbrio hidrostático, que:

\left(P(r) - P(r + dr)\right)dA = dmg(r)


-dP dA = \rho(r) dr dA g(r)


\dfrac{dP}{dr} = -g(r) \rho(r) = -\dfrac{GM}{r^2} \rho(r)


A pressão total P será a soma da pressão de radiação P_r = \dfrac{4 \sigma T^4}{3c} com a pressão do gás ideal P_G = \dfrac{\rho}{\mu}k_B T , logo:

 \dfrac{16\sigma}{3c}T^3(r)\dfrac{dT(r)}{dr} + \dfrac{d\rho(r)}{dr}\frac{k_BT(r)}{\mu} + \dfrac{\rho(r)}{\mu}k_B\frac{dT(r)}{dr} = -\dfrac{Gm(r)}{r^2} \rho(r)


Aplicando a equação acima pra fotosfera estelar, teremos que r = R, m(R) = M e, por aproximação, dr \approx \Delta r, sendo  \Delta r a espessura da atmosfera, dT(R) = \Delta T e, d\rho(R) = -\rho(R) (já que a densidade da estrela cai de \rho(R), na fotosfera, para 0, fora da atmosfera). Logo:

 \dfrac{16\sigma}{3c}T^3\dfrac{\Delta T}{\Delta r} - \dfrac{\rho(R)}{\Delta r}\dfrac{k_BT}{\mu} + \dfrac{\rho(R)}{\mu}k_B\dfrac{\Delta T}{\Delta r} = -\dfrac{GM}{R^2} \rho(R)


 \left(\dfrac{16\sigma}{3c}T^3 + \dfrac{\rho(R)}{\mu}k_B \right) \dfrac{\Delta T}{\Delta r} = \dfrac{\rho(R)}{\Delta r}\frac{k_BT}{\mu} - \dfrac{GM}{R^2} \rho(R)


 \Delta T = \dfrac{ \rho(R)\dfrac{k_BT}{\mu} - \dfrac{GM}{R^2}\rho(R) \Delta r}{\dfrac{16\sigma}{3c}T^3 + \dfrac{\rho(R)}{\mu}k_B }


Note que:

 \rho(R) \approx \dfrac{3M}{4\pi R^3}


Logo:

\Delta T = \dfrac{\dfrac{ 3M k_BT}{4\pi R^3\mu} - \dfrac{3GM^2}{4\pi R^5} \Delta r}{\dfrac{16\sigma}{3c}T^3 + \dfrac{3M}{4\pi R^3\mu}k_B }

Por fim, note que:

 I \propto T^4


 \Delta I \propto 4T^3 \Delta T


\dfrac{\Delta I}{I} = \dfrac{4 \Delta T}{T}


 \boxed{\dfrac{\Delta I}{I} =\dfrac{\dfrac{ 3M k_BT}{\pi R^3\mu} - \dfrac{3GM^2}{\pi R^5} \Delta r}{\dfrac{16\sigma}{3c}T^4 + \dfrac{3M}{4\pi R^3\mu}k_B T}}

[collapse]
Gabarito

a)  Demonstração.

b) Demonstração.

c)  Demonstração.

d)

 \boxed{\dfrac{\Delta I}{I} =\dfrac{\dfrac{ 3M k_BT}{\pi R^3\mu} - \dfrac{3GM^2}{\pi R^5} \Delta r}{\dfrac{16\sigma}{3c}T^4 + \dfrac{3M}{4\pi R^3\mu}k_B T}}

[collapse]